schwingrocker
Thanks Received: 0
Forum Guests
 
Posts: 23
Joined: July 01st, 2012
 
 
 

Q21 - Economics Professor: Marty's Pizza

by schwingrocker Sun Sep 16, 2012 11:57 pm

B is a mistaken reversal, correct? When diagraming the stimulus can we automatically assume that checker's motive was to hurt Marty's pizza is the necessary condition simply because it is the conclusion?
 
timmydoeslsat
Thanks Received: 887
Atticus Finch
Atticus Finch
 
Posts: 1136
Joined: June 20th, 2011
 
 
trophy
Most Thanked
trophy
First Responder
 

Re: Q21 - Economics Professor: Marty's Pizza

by timmydoeslsat Mon Sep 17, 2012 12:30 am

Yeah B is no good for us as it using a conditional statement with a sufficient condition about us having a company wishing to hurt a competitor. We want to know if we, in fact, have that situation occurring.

For this kind of sufficient assumption stimulus, you will often see the correct answer use the conclusion as its necessary condition due to the way the argument is structured. The author believes he has sufficient evidence to grant the conclusion's truth. We just need a conditional statement here to hurdle him over the finish line.
 
samuelfbaron
Thanks Received: 6
Elle Woods
Elle Woods
 
Posts: 71
Joined: September 14th, 2012
 
 
 

Re: Q21 - Economics Professor: Marty's Pizza

by samuelfbaron Sun May 19, 2013 3:07 pm

For the sake of review:

(A) If refusing to accept coupons --> Desire to hurt the competitor

Could this almost be considered a principle which the Economics professor assumes?

(B) Desire to hurt competitor --> Refuse to accept coupons

(B) Mixes up sufficient and necessary conditions.
User avatar
 
eliot_howington
Thanks Received: 0
Vinny Gambini
Vinny Gambini
 
Posts: 2
Joined: June 01st, 2014
 
 
 

Re: Q21 - Economics Professor: Marty's Pizza

by eliot_howington Sun Jun 01, 2014 8:30 pm

Can anyone explain to me why answer choice C is wrong? It includes the conclusion from the stimulus ("hurt the competitor"), which makes it seem like it could be a correct choice.
Is it wrong because it includes the fact that coupons were accepted by all other local competitors and that is outside the scope of the conclusion?

Thanks! :)
 
christine.defenbaugh
Thanks Received: 585
Atticus Finch
Atticus Finch
 
Posts: 536
Joined: May 17th, 2013
 
This post thanked 3 times.
 
 

Re: Q21 - Economics Professor: Marty's Pizza

by christine.defenbaugh Tue Jun 03, 2014 7:48 pm

Great question, eliot_howington!

This is a Sufficient Assumption question, and so we're going to need an answer choice that absolutely guarantees the validity of the conclusion. First stop: core breakdown!

We have a rather long and meandering story here - a tale of two pizza parlors. But the final upshot is this:

    PREMISE: 1) Checkers refused the coupons
    2) Accepting them would have cost Checkers nothing and would have satisfied some potential customers

    CONCLUSION: Checkers' motive must have been to hurt Marty's


Correct answers to Sufficient Assumption questions often follow a format of: If [premise], then [conclusion]. This type of conditional would guarantee the conclusion, once we know the premise - so it would absolutely guarantee that the argument works! Conditional statements that are even broader are great too, as long as they still guarantee the conclusion.

Here, (A) is giving us the classic pattern: If [company refuses to accept coupons even though it would satisfy potential customers], then [company's motivation is to hurt the competitor].

The problem with (C).
First, notice that this isn't actually a conditional statement. This is similar to (A) in that they both discuss refusing to accept coupons and the motivation being 'to hurt the competitor'. But (A) was a rule (conditional) that applied to ALL companies that refused coupons in certain circumstances. As a result, it was a guarantee of that company's motivation. (C), in contrast, is only telling us that there is at least one company in the world for whom this is true.

Do we know that Checkers is in that category? No! This doesn't guarantee the motivation for Checkers at all!

Let's review the remaining incorrect answers as well:
(B) As samuelfbaron and timmydoeslsat lay out above, this is a reversal of the conditional we need to guarantee the conclusion.

(D) This might be distilled as If [accept coupons] then [help competitor]. This tells us about companies that DO accept coupons - we need to know about companies that DON'T. Also, the result here is about actually helping a competitor, while our conclusion is about motivation.

(E) Again, the if-trigger is not our situation, but instead follows what happens if accepting would not enable satisfying customers. Also, we need a result that guarantees 'motivation = hurt competitor', and this instead results in 'motivation = satisfy customers'.

Remember, the classic pattern for Sufficient Assumptions is "If [premise], then [conclusion]." Any blanket statement that covers this idea will do to guarantee the conclusion! While not all Sufficient Assumptions follow this format, it's common enough that it pays to be familiar with it!

Does that help clear this question up a bit?
 
jasonleb1
Thanks Received: 1
Jackie Chiles
Jackie Chiles
 
Posts: 32
Joined: April 09th, 2015
 
 
 

Re: Q21 - Economics Professor: Marty's Pizza

by jasonleb1 Wed Sep 16, 2015 3:05 pm

I had some difficulty with this one since, at least to me, it didn't seem like a normal sufficient assumption question.

I knew going into it that I had to guarantee that there could be no other motivation for denying those coupons besides hurting its competitor (given that accepting them cost them nothing and would satisfy their customers). However, I eliminated A - and then every other answer choice so I just guessed BB - because I thought it was a necessary assumption, not a sufficient assumption.

The argument gives us two premises, that it would cost them nothing AND that it would have satisfied customers thereby leaving no other motivation than to hurt the competitor. But A only gets us half way by making that guarantee for the satisfied customers part. What if accepting the coupon would have cost him a ton of money? A doesn't account for that and it is a valid reason for rejection as given by our premises:

notCost AND Satisfy --> Hurt

notHurt --> Cost OR notSatisfy

So, at least to me, that makes it impossible for A to be sufficient to guarantee the outcome.

Any insight?